You are on page 1of 9

Analytical Reasoning

Analytical reasoning questions overtax peoples’ short term memories .Researchers supports the belief that the human brain can keep
between five to nine items of information active & readily available at any moments. That is why aids to short term memory ,external
memory , in the form of abbreviated written rules, diagrams, maps, networks, etc., are helpful in solving these problems.
The initial conditions given in an analytical problem can be thought of as rules. Some rules may be modified in the set of questions
which follow the problem. These rules fall into four general classes:
1. Requirements: “A group must contain three members”.
2. Prohibition : “ A group may contain three members”.
3. Allowances: “ A group may contain from three to five members”.
4. Conditionals: “ if any group contains three members then no other group may contain five members”.
These rules can be additionally & independently analyzed into at least eight categories.
(i) Sizing rules: Sizing rules are concerned with the number of elements or members in sets and subsets of the problem.
(ii) Labeling rules: “ A group of participants , Ali, Bushra, Maazin, Rayyan & Saad are in a contest”.In this case it will often simplify
the solutions to abbreviate the labels as A,B,M, R & S or 1,2, 3, 4 & 5.
(iii) Frequency rules: “Ali must be in one of the contest”. Ali & Bushra must not compete together morethan twice”.
Ali may be in all rounds of the contest. If Ali is in any round then Bushra may not be in more than two rounds”. Notice how the four
classes of rules are exemplified in these frequency rules.
(iv) Grouping rules: “ A must compete with B”. “ B must not compete with M”. “ M may compete with A ,R & S”.” If M competes then A
must also compete”. These rules may also be referred to as scheduling rules, although analytical problems which concern scheduling of
events often combine sizing, frequency & grouping rules.
(v) Ordering rules: “A must be next to B”. “ B may not follow M or R”. “ M may be above or below R”..If R is to the right of S then A
must be to S’s left.
(vi) Interaction rules: problems which specify relations between two or more sets of categories or dimensions are often most clearly
represented in matrix or table form, whether they present requirements , prohibition or allowances. In the contestants
,A through S, must compete in the following rounds of the contest shown in the table.
Rounds A B M R S
1
2
3
4

Examples:
Read the following information carefully and answer the questions given below it.
(i) Six friends , P,Q,R,S,T & U are member of a club and play a different game of Football ,Cricket, Tennis ,Basketball
Badminton & Volleyball.
(ii) T who is taller than P & S plays Tennis.
(iii) The tallest among them plays basketball.
(iv) The shortest among them plays volleyball.
(v) Q& S neither play Volleyball nor Basketball.
(vi) R plays Volleyball.
(vii) T is between Q who plays Football & P in order of height.
1. What does S play?
(a) Football (b) Either cricket or badminton (c) Cricket (d) Badminton (e) none of these
2. Who among them is taller than R but shorter than P?
(a) T (b) Data inadequate (c) Q (d) U (e) None of these
3. Who among them plays Basketball?
(a) S (b) U (c) Q (d) R (e) None of these
4. Which of the following statement is not true?
(a) T is taller than R (b) U is taller than Q (c) P is shorter than R
(d) Q is taller than S (e) S is taller than R
5. Who will be at the third place if they are arranged in descending order of their height.
(a) T (b) Q (c) P (d) S (e) None of these

Players Games
Solution
U Basketball
Q Football
T Tennis
P Cricket or badminton
S Badminton or cricket
R Volleyball
2. Miss white holds a spelling bee each Friday morning for eight of her best spellers. The first students eliminated is ranked 8 th
& the victor is ranked 1st.The weekly records show the following characteristics.
Nikko is always first.
Liz is always right before Patrice.
Jamal is always somewhere ahead of Quincy.
One student is always between Kelly & Ophelia with either of them being the better one.
Mariana & Patrice are always better than Jamal .
1. Of the following records which could be from Miss White’s Class?
(a) Nikko , Liz, Patrice ,Jamal , Kelly, Mariana, Quincy, Ophelia.
(b) Nikko , Liz, Jamal, Patrice, Mariana, Kelly, Quincy, Ophelia.
(c) Nikko, Mariana, Jamal, Liz, Patrice, Kelly, Quincy, Ophelia.
(d) Nikko , Liz, Patrice, Quincy, Mariana, Kelly, Jamal, Ophelia.
(e) Nikko , Liz, Patrice, Mariana, Jamal , Kelly, Quincy, Ophelia.
2. On one record Patrice is 4 th & Ophelia is 5 th .which of the following must also be true of that record?
(a) Mariana is 3rd. (b) Jamal is 2nd (c) Kelly is 3rd (d) Jamal is 7 th (e) Quincy is 8th
th
3. Which student must be 7 in a record in which Kelly is ahead of Mariana?
(a) Jamal (b) Kelly (c) Mariana (d) Quincy (e) Ophelia
Solution
First we list the spellers.
J K L M N P O Q
Then we simplify the rules.
(i) N always first.
(ii) L &P always paried with L first.
(iii) J always ahead of Q.
(iv) K & O always have one between them.
(v) M & P always ahead of J.
1. This is a could be question.
Option a is incorrect b/c its given that M should be ahead of J.
Option b is incorrect b/c its given that L should be right before P.
Option c is incorrect b/c its given that P should be ahead of J.
Option d is incorrect b/c its given that P should be ahead of J.
E is the correct option.
2. A diagram may be useful to summarize that we know .Based on the information given, we have:
N M L P O J K Q
Looking at the alternatives we see that a, b, c & d do not fit the ordering. So e is the correct answer.
3. N L P K M O P Q N K M O L P J Q
If K precedes M & all the other conditions are met, he first 6 places are N, followed by one of two ordering
O , K, L , M,O, P. since J is always ahead of Q . J must be in 7 th place. Looking at the alternatives ,we choose (a)
Passage for Question 1
A medical clinic has a staff of five doctors—Drs. Albert, Burns, Calogero, Defeo, and Evans. The national medical
society sponsors exactly five conferences, which the clinic’s doctors attend, subject to the following constraints:
If Dr. Albert attends a conference, then Dr. Defeo does not attend it.
If Dr. Burns attends a conference, then either Dr. Calogero or Dr. Defeo, but not both, attends it.
If Dr. Calogero attends a conference, then Dr. Evans does not attend it.
If Dr. Evans attends a conference, then either Dr. Albert or Dr. Burns, but not both, attends it.
Question 1
If Dr. Burns attends one of the conferences, then which one of the following could be a complete and accurate list of
the other members of the clinic who also attend that conference?
(A) Drs. Albert and Defeo
(B) Drs. Albert and Evans
(C) Drs. Calogero and Defeo
(D) Dr. Defeo
(E) Dr. Evans
Explanation for Question 1
This question requires you to determine, from the conditions given, which doctors can attend the same conferences.
The question tells us that “Doctor Burns attends one of the conferences,” and we are asked to choose the response
that could be a list of all and only those doctors who attend the conference with Dr. Burns. Since we are asked what
could be a “complete and accurate list” [emphasis added] of those doctors who attend the conference with Dr.
Burns, we can eliminate as incorrect those responses which either are inaccurate (that is, cannot be true), or
incomplete (that is, do not include everyone who must accompany one or more of the doctors going to the
conference). This can be determined easily without the use of a diagram.
Response (A) states that, along with Dr. Burns, Drs. Albert and Defeo also attend the conference. But the first
condition tells us that “if Dr. Albert attends a conference, then Dr. Defeo does not attend it.” So, Drs. Burns, Albert,
and Defeo cannot all attend the same conference. Response (A), then, is incorrect.
Response (B) is incorrect for a similar reason. The fourth condition tells us what must be true if Dr. Evans attends a
conference, namely, that “either Dr. Albert or Dr. Burns, but not both, attends it.” Since we know that Dr. Burns
attends the conference, we know that it cannot be true that both Drs. Albert and Evans also attend that conference.
Response (C) is also incorrect. The second condition tells us what must be true if Dr. Burns attends a conference.
Since we know that Dr. Burns does attend the conference, we also know that “either Dr. Calogero or Dr. Defeo, but
not both, attends it.”
Responses (D) and (E) must be evaluated slightly differently. No condition rules out Dr. Burns’s and Dr. Defeo’s
going to the same conference—response (D)—and no condition forbids Dr. Evans’s going with Dr. Burns to a
conference—response (E). But recall that the question asks for what could be a “complete and accurate list” of the
doctors who attend the conference with Dr. Burns. We know from the second condition that at least one other person
must accompany Dr. Burns, and that among those who accompany Dr. Burns are either Dr. Calogero or else Dr.
Defeo. Since the conditions do not require anyone to accompany Dr. Defeo, it is possible that Dr. Defeo is the only
person to accompany Dr. Burns. Thus, response (D) is an accurate response, in that it is possible that Drs. Burns and
Defeo attend the same conference, and it is a complete response, in that Drs. Burns and Defeo could be the only
doctors of the five to attend the conference. So response (D) is correct.
Response (E) is incorrect because we know that if Dr. Burns goes, someone other than Dr. Evans must also go.
Response (E) then is incomplete. It fails to list at least one doctor whom we know must also accompany Dr. Burns.
This question is classified as “moderately difficult.”
Passage for Questions 2 and 3
Seven piano students—T, U, V, W, X, Y, and Z—are to give a recital, and their instructor is deciding the order in
which they will perform. Each student will play exactly one piece, a piano solo. In deciding the order ofperformance, the instructor
must observe the following restrictions:
X cannot play first or second.
W cannot play until X has played.
Neither T nor Y can play seventh.
Either Y or Z must play immediately after W plays.
V must play either immediately after or immediately before U plays.
Question 2
If V plays first, which one of the following must be true?
(A) T plays sixth.
(B) X plays third.
(C) Z plays seventh.
(D) T plays immediately after Y.
(E) W plays immediately after X.
Explanation for Question 2
This question deals with an ordering relationship defined by a set of conditions as to when the seven piano students
will perform. As an aid in visualizing this problem we can draw a simple diagram that shows the seven recital slots
arranged in order from left to right. Student V is shown in the first slot, as specified by the condition that “V plays
first”:We can immediately fill in one of the empty slots in the diagram.

The condition that “V must play either


immediately after or immediately before U plays” tells us that U must occupy the second slot in the recital schedule.
This is shown below: Since the question asks us what must be true, we can eliminate incorrect responses by showing that they could be
false without violating the conditions. Response (A) is incorrect because the statement that “T plays sixth” is not necessarily true—we
can place T in one of the slots other than sixth and still meet all the conditions of the problem One such recital schedule, with T
playing third, is shown in the diagram below:

We can develop this schedule as follows. With V, U, and T in the first three positions, there are four positions left
for W, X, Y, and Z.
_ W must come after X—because of the condition that “W cannot play until X has played”—so if we put X fourth
and W fifth, this condition will be met.
_ This leaves two possible slots for Y and Z. Y cannot play seventh because of the condition that “Neither T nor
Y can play seventh,” so we will place Y sixth and Z seventh.
A check will verify that this schedule meets the given conditions of the problem, including the one that “Either Y or
Z must play immediately after W plays.”
The schedule shown in the diagram also demonstrates that response (B) is incorrect. In it, X plays fourth, so it is not
correct that the statement, “X plays third,” must be true .Response (C), “Z plays seventh,” is the credited response. We can show this
by demonstrating that:
_ all the conditions can be met with Z in the seventh slot, and
_ some of the conditions would be violated with Z in any slot other than seventh.
To demonstrate that Z can play seventh, we can refer to the schedule that was developed for the discussion of
response (A), above. In it, Z plays seventh, and all the conditions in the problem are met.
To demonstrate that Z cannot play in a slot other than seventh, we can attempt to find another student to play
seventh. We already know that neither U nor V can play seventh. Hence, there are four remaining players: T, W, X,
and Y. However, a review of the given conditions shows that none of those players can play seventh:
_ The third condition states that “Neither T nor Y can play seventh.”
_ W can’t play seventh, because there must be a slot following W’s in order to meet the condition, “Either Y or Z
must play immediately after W plays.” If W plays seventh, then there is no such slot left for Y or Z.
_ For a similar reason X can’t play seventh, because there must be a slot following X’s in order to meet the
condition, “W cannot play until X has played.”
Since Z can play seventh and no other player can, then the statement that Z must play seventh is correct and (C) is
the credited response.
Response (D) is incorrect because it is not necessarily true that “T plays immediately after Y.” In our discussion of
response (A), we developed a schedule in which T plays third and Y plays sixth, yet all conditions are satisfied.
Response (E) is incorrect because, as shown in the diagram below, it is not necessarily true that “W plays
immediately after X.” This schedule is obtained by simply reversing the order of players W and Y in the schedule
we developed in the analysis of response (A).
A review will show that all of the given conditions are met by this schedule
.This question is classified as “difficult.”
Question 3
If U plays third, what is the latest position in which Y can play?
(A) first (B) second (C) fifth (D) sixth (E) seventh
Explanation for Question 3
This question involves the same original conditions as the previous problem, but it begins with a different
assumption: “U plays third.” The test taker must determine what effect this assumption would have on the possible
positions in which Y can appear in the recital schedule. The correct response is (D), because student Y can play as late as sixth under
the given constraint that “U plays third.” The diagram below shows a recital order that meets all the given conditions and has Y
performing in the sixth position.

One strategy for arriving at this solution is to work backward to see which
position is the latest in which we can place Y and at the same time produce a
recital schedule that meets all the given conditions .Using that approach, we
immediately see that Y cannot play as late as seventh, because of the condition
that“Neither T nor Y can play seventh.” Backing up and placing Y sixth,
we can begin to fill in the schedule, as follows:

This schedule has five empty slots, into which we must fit players T, V, W, X, and Z. The following is a series of
reasoning steps that can be used:
_ From our analysis of the previous question, we know that players T, W, and X cannot play seventh, but that Z
can, so we can tentatively place Z in the seventh slot.
_ We also know that “Either Y or Z must play immediately after W plays.” If we place W in the fifth slot, this
condition will be met.
_ By placing V in the second slot, we can meet the condition that “V must play either immediately after or
immediately before U plays.”
_ We must place the remaining two players, T and X, in the two remaining slots, the first and the fourth. Because
the first condition states that “X cannot play first … ,” we will place X in the fourth slot and T in the first. These
positions will meet the conditions that apply to T and X: T will avoid playing seventh and X will play before W.
Since Y can play as late as sixth, response (D) is the correct solution.
This question is classified as “moderately difficult.”
Passage for Questions 4 and 5
On a particular Saturday, a student will perform six activities— grocery shopping, hedge trimming, jogging, kitchen
cleaning, laundry, and motorbike servicing. Each activity will be performed once, one at a time. The order in which
the activities are performed is subject to the following conditions:
Grocery shopping has to be immediately after hedge trimming.
Kitchen cleaning has to be earlier than grocery shopping.
Motorbike servicing has to be earlier than laundry.
Motorbike servicing has to be either immediately before or immediately after jogging.
Question 4
If laundry is earlier than kitchen cleaning, then hedge trimming must be
(A) fifth (B) fourth (C) third (D) second (E) first
Explanation for Question 4
This problem is concerned with determining the order in which six activities will be performed. As with many
questions involving relative ordering or ranking, it is likely that you will find it useful to diagram the various
relationships given in the passage.
The first condition in the passage tells us that grocery shopping has to be immediately after hedge trimming, which
we can abbreviate as follows: 1. HG
The second condition tells us that kitchen cleaning has to be earlier than grocery shopping, which we can abbreviate
as follows, where “…” is used to represent “earlier than” (which means any time before, including immediately before):
2. K…G
The third condition tells us that motorbike servicing has to be earlier than laundry, and the fourth condition tells us
that motorbike servicing has to be either immediately before or immediately after jogging. These conditions can be
abbreviated as follows, where the / symbol is used to represent “or”:
3. M…L
4. MJ/JM
Notice that the information specified in these four conditions can be collapsed into two ordering statements:
I. K…HG (first and second conditions)
II. MJ/JM…L (third and fourth conditions)
Question 7 introduces the new supposition “laundry is earlier than kitchen cleaning”:L…K
This new supposition works to further collapse the ordering statements in I and II to the single statement below; that
is, if L must be earlier than K, then we know that the activities must be ordered like this:
MJ/JM…L…K…HG
So, with the addition of the new supposition, there are exactly two possible orderings of the six activities, differing
only with respect to whether motorbike servicing is immediately before or immediately after jogging:
1 2 3 4 5 6
MJLKHG
JMLKHG
Question 7 asks what position hedge trimming must be in, given the new supposition. What we see here is that
hedge trimming must be the fifth activity performed, and so answer choice (A) is correct.
This question is classified as “easy.”
Question 5
Which one of the following, if substituted for the condition that motorbike servicing has to be earlier than laundry,
would have the same effect in determining the order of the student’s activities?
(A) Laundry has to be one of the last three activities.
(B) Laundry has to be either immediately before or immediately after jogging.
(C) Jogging has to be earlier than laundry.
(D) Laundry has to be earlier than hedge trimming.
(E) Laundry has to be earlier than jogging.
Explanation for Question 5
This question asks you to select the condition which, if substituted for the third condition in the passage (repeated
below), would have the same effect as the original condition.
Third condition: Motorbike servicing has to be earlier than laundry.
In this case, you can deduce that the correct answer choice is (C):
(C) Jogging has to be earlier than laundry.
The fourth condition in the passage tells you that motorbike servicing has to be either immediately before or
immediately after jogging. That is, M and J must be ordered as a block, either MJ or JM, with respect to the other
four activities. Thus, if, as the original third condition states, M has to be earlier than L, then we know that J must
also be earlier than L. Conversely, if, as the new condition in answer choice (C) states, J has to be earlier than L,
then we know that M must also be earlier than L. In short, the third condition and answer choice (C) have exactly
the same effect. Therefore, answer choice (C) is correct.
Another way to approach this kind of question is to attempt to eliminate all of the incorrect answer choices. Under
this approach, you want to rule out any answer choice that does either of the following:
_ rules out outcomes that the original condition allows
_ allows outcomes that the original condition rules out
Let’s see how this approach would enable us to eliminate answer choices (A), (B), (D), and (E).
Consider the condition presented in answer choice (A):
(A) Laundry has to be one of the last three activities.
We can first ask whether this condition would rule out outcomes that the original third condition allows. To answer
this question, we must simply determine whether there is an outcome allowed by the original third condition along
with the other conditions in which laundry is one of the first three activities. Here is such an outcome:
1 2 3 4 5 6
MJL KHG
Because the original third condition allows this outcome, but the condition in answer choice (A) does not, answer
choice (A) cannot be correct.
Consider answer choice (B):
(B) Laundry has to be either immediately before or immediately after jogging.
Again, we want to first determine whether this new condition would rule out outcomes that the original third
condition allows. To answer this question, we must simply determine whether there is at least one outcome allowed
by the original third condition along with the other conditions in which laundry is neither immediately before nor
immediately after jogging. Here is one such outcome:
1 2 3 4 5 6
KHGJML
This outcome, although allowed by the original third condition, would be ruled out by the alternative condition
given in answer choice (B). Thus, answer choice (B) cannot be correct.
Next consider answer choice (D):
(D) Laundry has to be earlier than hedge trimming.
Again, we want to first determine whether this new condition would rule out outcomes that the original third
condition allows. To answer this question, we must simply determine whether there is at least one outcome allowed
by the original third condition along with the other conditions in which laundry is not earlier than hedge trimming.
One such outcome was given immediately above: since L is not earlier than H in this outcome, it would be ruled out
by the condition in answer choice (D). So, answer choice (D) rules out an outcome that the original third condition
allows, and therefore (D) cannot be the correct answer choice.
Finally, consider answer choice (E):
(E) Laundry has to be earlier than jogging.
Again, we want to first determine whether having this new condition would rule out outcomes that are allowed when
the original third condition is in place. To answer this question, we must simply determine whether there is at least
one outcome allowed by the original third condition along with the other conditions in which laundry is not earlier
than jogging. One such outcome was given above: since L is not earlier than J in this outcome, it would be ruled out
Sample Questions with Explanations for LSAT—India 8
by the condition presented in answer choice (E). So, answer choice (E) rules out an outcome that the original third
condition allows, and therefore (E) cannot be the correct answer choice.
In sum, answer choices (A), (B), (D), and (E) can all be eliminated because in each case the condition is one that
rules out outcomes that the original condition allows. For this particular question, there was no need to consider
whether any of the options could be eliminated because they allowed outcomes that the original condition ruled out.
This question is classified as “moderately difficult.”
Practice
1.Four friends in the sixth grade were sharing a pizza. They decided that the oldest friend would get the extra piece. Randy is two
months older than Greg, who is three months younger than Ned. Kent is one month older than Greg. Who should get the extra piece of
pizza?
a. Randy b. Greg c. Ned d. Kent
2.Martina is sitting in the desk behind Jerome.Jerome is sitting in the desk behind Bryant. Bryant is sitting in the desk behind
Martina.If the first two statements are true, the third statement is
a. true. b. false. c. uncertain.
3.Fact 1: Jessica has four children.Fact 2: Two of the children have blue eyes and two of the children have brown eyes.Fact 3: Half of
the children are girls.If the first three statements are facts, which of the following statements must also be a fact?
I. At least one girl has blue eyes.
II. Two of the children are boys.
III. The boys have brown eyes.
a. II only b. I and III only c. II and III only d. None of the statements is a known fact
4. A four-person crew from Classic Colors is painting Mr. Field’s house. Michael is painting the front of the house. Ross is in the alley
behind the house painting the back. Jed is painting the window frames on the north side, Shawn is on the south. If Michael switches
places with Jed, and Jed then switches places with Shawn, where is Shawn?
a. in the alley behind the house b. on the north side of the house
c. in front of the house d. on the south side of the house
Answer questions (i) –(ii) on the basis of the information below.
5. A weekly television show routinely stars six actors,J, K, L, M, N, and O. Since the show has been on
the air for a long time, some of the actors are good friends and some do not get along at all. In an effort to keep peace, the director
sees to it that friends work together and enemies do not. Also,as the actors have become more popular, some of them need time off to
do other projects. To keep the schedule working, the director has a few things she must be aware of:
■ J will only work on episodes on which M is working.
■ N will not work with K under any circumstances.
■ M can only work every other week, in order to be free to film a movie
■ At least three of the actors must appear in every weekly episode.
(i) In a show about L getting a job at the same company J already works for and K used to work for, all three actors will appear. Which
of the following is true about the other actors who may appear?
a. M, N, and O must all appear. b. M may appear and N must appear.
c. M must appear and O may appear. d. O may appear and N may appear.
e. Only O may appear.
(ii). Next week, the show involves N’s new car and O’s new refrigerator. Which of the following
is true about the other actors who may appear?
a. M, J, L, and K all may appear. b. J, L, and K must appear.
c. Only K may appear. d. Only L may appear.
e. L and K must appear.
Answer questions (i) –(iii) on the basis of the information below.
6. Evan is a waiter in a café. After he turns in orders for the six people sitting at the counter—each of
whom is eating alone and is sitting in chairs numbered 1 through 6—the cook opens a window in
the kitchen and the order slips get messed up.Here’s what Evan remembers about the orders:
■ The entree orders are: fried eggs, a hamburger, a cheeseburger, a vegetable burger, soup, and a ham sandwich.
■ The two people who did not order sandwiches are sitting at chairs 3 and 4.
■ The person who ordered the cheeseburger and the one who ordered the hamburger are notsitting next to each other.
■ The person in chair number 5 is a regular. She will not sit next to anyone who is eating ham.
■ The person eating the vegetable burger is not sitting in chair 2, but is sitting between the person who ordered fried eggs and the
one who ordered a cheeseburger.
■ The customer who ordered the hamburger is not sitting next to the customer who ordered soup.
(i) To which customer should Evan serve the vegetable burger?
a. the customer in chair 2 b. the customer in chair 3
c. the customer in chair 4 d. the customer in chair 5
e. the customer in chair 66
(ii)To which customer should Evan serve the soup?
a. the customer in chair 1 b. the customer in chair 2
c. the customer in chair 3 d. the customer in chair 4
e. the customer in chair 5
(iii) To which customer should Evan serve the ham sandwich?
a. the customer in chair 1 b. the customer in chair 2
c. the customer in chair 4 d. the customer in chair 5
e. the customer in chair 6
Use the additional information below, along with the information before question 6(i-iii) , to answer questions (iv,v)
Now Evan has to decide who gets which side dish. Here is what he remembers, in addition to the previous information, about the
orders, which were: cole slaw, hash browns, onion rings, potato salad, french fries, and lettuce salad.
■ The person who ordered soup did not order french fries, hash browns, onion rings, or a,lettuce salad.
■ The person who ordered onion rings is sitting in chair 6.
■ The person who ordered potato salad is sitting between the person who ordered cole slaw and the one who ordered hash browns.
■ The person who ordered a vegetable burger ordered a lettuce salad.
■ The hash browns were ordered by the customer who ordered fried eggs.
(iv) With which entrée does the potato salad belong?
a. soup b. hamburger c. cheeseburger d. fried eggs e. ham sandwich
(v) With which entrée do the French fries belong?
a. soup b. cheeseburger c. hamburger d. fried eggs e. ham sandwich
Answers:
1. Ned is oldest one.
2. Given the information in the first two statements,Bryant is sitting in front of both Jerome and Martina, so the third statement
must be false.
3. Since one-half of the four children are girls,two must be boys. It is not clear which children have blue or brown eyes.
4. After all the switches were made, Shawn is in front of the house. Ross is in the alley behind
the house, Michael is on the north side, and Jed is on the south.
5. (i) . J will only work in episodes in which M is working and there are no restrictions on O’s schedule.
However, N will not work with K, so M must appear and O may appear.
(ii).d. K will not work with N, so choices c and e are incorrect. M can only work every other week, so
choice a is incorrect. Since M is not working, J will not work, so choice b is incorrect.
6.(i) The person who ordered potato salad cannot be in chair 1 or 6, since he is sitting between two
people. The person who ordered fried eggs ordered hash browns and is sitting in chair 4.he person who ordered potato salad is on one
side of chair 4, either 3 or 5. He cannot be in chair 5 and still be next to both the hash browns and the cole slaw, so he must be in chair
3, which is where the soup was ordered.
(ii) c. If the potato salad is with the soup and the hash browns are with the fried eggs, then the cole slaw must be with the ham
sandwich, in chairs 2, 3, and 4. The lettuce salad is with the vegetable burger in chair 5. The onion rings belong to the cheeseburger in
chair 6, leaving the french fries for the hamburger in chair 1.
––Exercise Exercise
1. The office staff of the XYZ Corporation presently consists of three bookkeepers (L, M andN ) and five secretaries (O, P, Q, R and
S). Management is planning to open a new officein another city sending three secretaries and two bookkeepers from the present staff.
To do so they plan to separate certain individuals who do not function well together. The following guidelines were established to set up
the new office:
(a) Bookkeepers L and N are constantly finding faults with one another therefore should not be sent together to the new office.
(b) N and P function well alone but not as a team. They should be separated.
(c) O and R have not been on speaking terms for many months. They should not go together.
(d) Since O and Q have been competing for a promotion, they should not be in one team.
Based on the information given above find the correct answers to the following Questions:
1. If M insists on staying back then how many combinations are possible?
A. 1 B. 2 C. 3 D. None
2. If L is to be moved as one of the bookkeepers, which of the following CANNOT be a possible working unit?
A. LMOPS B. LMPQS C. LMORS D. LMPRS
3. If N is sent to the new Office which member of the staff CANNOT be sent?
A. O B. M C. Q D. R
4. If O is sent to the new office then which of the following is a possible team?
A. LMOPR B. MNOQS C. MNOPS D. LMOPS
5. If both N and Q are moved to the new office, how many combinations are possible?
A. 2 B. 3 C. 4 D. 1
2. A map representing countries R, S, W, X, Y and Z is to be drawn. Adjacent countries cannot have the same color in the map. The
countries adjacent to each other are as follows:
Each of R, S, X and Y is adjacent to W. X is adjacent to Y.Each of R and S is adjacent to Z.
If X is the same color as Z then it must be true that
A. W is a different color from any other country.
B. S is a different color from any other country.
C. X is the same color as Y.
D. S is the same color as X.
Two statements, labeled I. & II, follow each of the following questions. The statements contain certain information. In the
questions you do not actually have to compute an answer, rather you have to decide whether the information given in the
statements I. and II. is sufficient to find a correct answer by using basic mathematics and every day facts?
3. A long distance runner has just completed running 28 miles. How long did it take him to finish the journey?
I. His record speed is 8.25 miles per hour.
II. His average speed through the journey was 8 miles per hour.
A. Statement I. ALONE is sufficient but II. ALONE is not sufficient to answer this question.
B. Statement II. ALONE is sufficient but I. ALONE is not sufficient to answer this question.
C. Statements I. and II. TOGETHER are sufficient to answer the question but NEITHER of them is sufficient ALONE.
D. Statements I. and II. COMBINED are NOT sufficient to answer the question and additional information is needed to find the
correct answer.
4. Captain of the national cricket team has to be the most popular member of the team.Who is the captain of Pakistan’s national cricket
team?
I. Waqar is the best player on the team.
II. Waseem is the senior-most member.
A. Statement I. ALONE is sufficient but II. ALONE is not sufficient to answer this question.
B. Statement II. ALONE is sufficient but I. ALONE is not sufficient to answer this question.
C. Statements I. and II. TOGETHER are sufficient to answer the question but NEITHER of them is sufficient ALONE.
D. Statements I. and II. COMBINED are NOT sufficient to answer the question and additional information is needed to find the
correct answer.
5. In a BCE class at CIIT, 30 boys and 10 girls registered Calculus II. How many boys passed the course?
I. 5 students could not pass.
II. There were 2 girls who obtained A grade.
A. Statement I. ALONE is sufficient but II. ALONE is not sufficient to answer this question.
B. Statement II. ALONE is sufficient but I. ALONE is not sufficient to answer this question.
C. Statements I. and II. TOGETHER are sufficient to answer the question but NEITHER of them is sufficient ALONE.
D. Statements I. and II. COMBINED are NOT sufficient to answer the question and additional information is needed to find the
correct answer.
6. A horse ran 100 miles without stopping. What was its average speed in miles per hour?
I. The journey started at 8 PM and ended at 4 AM the following day.
II. The horse ran 20 miles per hour for the first 50 miles.
A. Statement I. ALONE is sufficient but II. ALONE is not sufficient to answer this question.
B. Statement II. ALONE is sufficient but I. ALONE is not sufficient to answer this question.
C. Statements I. and II. TOGETHER are sufficient to answer the question but NEITHER of them is sufficient ALONE.
D. Statements I. and II. COMBINED are NOT sufficient to answer the question and additional information is needed to find the
correct answer.
7. How much time will computer a need to solve 150 problems?
I. The computer needs 50 seconds to solve one problem.
II. Computer never takes more than 60 seconds to solve a problem.
A. Statement I. ALONE is sufficient but II. ALONE is not sufficient to answer this question.
B. Statement II. ALONE is sufficient but I. ALONE is not sufficient to answer this question.
C. Statements I. and II. TOGETHER are sufficient to answer the question but NEITHER of them is sufficient ALONE.
D. Statements I. and II. COMBINED are NOT sufficient to answer the question and additional information is needed to find the
correct answer.

You might also like